Use the Limit Comparison Test to prove convergence or divergence of the infinite series. sum_{n=2}^inftyfrac{n}{sqrt{n^3+1}}

UkusakazaL

UkusakazaL

Answered question

2021-01-31

Use the Limit Comparison Test to prove convergence or divergence of the infinite series.
n=2nn3+1

Answer & Explanation

Arnold Odonnell

Arnold Odonnell

Skilled2021-02-01Added 109 answers

Let the given expression be represented as,
an=nn3+1
Consider another function bn=1n
According to the limit comparison test-
1. If limn(anbn)= finite positive value, then series nan and nbn behave alike, i.e. either both converge or both diverge.
2. If limn(anbn)=0 and nbn converges, then series nan will also converge.
3. If limn(anbn) diverges, then series nan will also diverge.
Clearly the series n=1bn is a converging series as the term limn1n0
Check fot he series (anbn) as,
limn(anbn)=(nn311n)
=limn(nn31×n)
=limn(n3n31)
=limn(n3n31)
=limn(111n3)
As n, then 1n30,
limn(anbn)=110
=1
Since limn(anbn) is a finite positive value and n=1bn is converging, therefore the limit comparison test implies that, the series n=1an is also converging.
Consider the following,
n=2nn3+1=n=1nn3+1[nn3+1]n=1
=n=1nn3+1113+1
=n=1nn3+112
Consider that the difference between two finite values is also finite.
Thus, it is concluded that the series =n=2nn3+1 is converging.
Jeffrey Jordon

Jeffrey Jordon

Expert2021-12-27Added 2605 answers

Answer is given below (on video)

Do you have a similar question?

Recalculate according to your conditions!

Ask your question.
Get an expert answer.

Let our experts help you. Answer in as fast as 15 minutes.

Didn't find what you were looking for?